Download as pdf or txt
Download as pdf or txt
You are on page 1of 34

SIP Polity TEST 01

DO NOT OPEN THIS TEST BOOKLET UNTIL YOU ARE TOLD TO DO SO


Test Booklet Series

A
TEST BOOKLET
SIP
Polity

Maximum Marks: 100 Time Allowed: ONE HOUR


INSTRUCTIONS
1. IMMEDIATELY AFTER THE COMMENCEMENT OF THE EXAMINATION, YOU
SHOULD CHECK THAT THIS TEST BOOKLET DOES NOT HAVE ANY
UNPRINTED OR TORN OR MISSING PAGES OR ITEMS ETC. IF SO, GET IT
REPLACED BY A COMPLETE TEST BOOK.
2. Please note that it is the candidate’s responsibility to encode and fill in
the Roll Number and Test Booklet Series A, B, C or D carefully without any
omission or discrepancy at the appropriate places in the OMR Answer
Sheet. Any omission/discrepancy will render the Answer Sheet liable for
rejection.
3. You have to enter your Roll Number on the test
booklet in the Box provided alongside. DO NOT
write anything else on the Test Booklet.
4. This Test Booklet contains 50 items (questions). Each item is printed in
English. Each item comprises four responses (answers). You will select the
response which you want to mark on the Answer Sheet. In case you feel that
there is more than one correct response, mark the response which you consider
the best. In any case, choose ONLY ONE response for each item.
5. You have to mark all your responses ONLY on the separate Answer Sheet
provided.
See directions in the Answer Sheet.
6. All items carry equal marks.
7. Before you proceed to mark in the Answer Sheet the response to various items
in the Test Booklet, you have to fill in some particulars in the Answer Sheet as
per instructions sent to you with your Admission Certificate.
8. After you have completed filling in all your responses on the Answer Sheet and
the examination has concluded, you should hand over to the Invigilator only the
Answer Sheet. You are permitted to take away with you the Test Booklet.
9. Sheets for rough work are appended in the Test Booklet at the end.
10. Penalty for wrong answers:
THERE WILL BE PENALTY FOR WRONG ANSWERS MARKED BY A
CANDIDATE IN THE OBJECTIVE TYPE QUESTION PAPERS.
(i) There are four alternatives for the answer to every question. For each
question for which a wrong answer has been given by the candidate, one-
third of the marks assigned to that question will be deducted as penalty.
(ii) If a candidate gives more than one answer, it will be treated as wrong
answer even if one of the given answers happens to be correct and there
will be same penalty as above to that question.
(iii) If a question is left bank i.e., no answer is given by the candidate, there
will be no penalty for that question.

DO NOT OPEN THIS TEST BOOKLET UNTIL YOU ARE TOLD TO DO SO

1-A
SIP Polity TEST 01

1. How many of the pairs given below is/are 3. The Indian Constitution embodies the
correctly matched? negative concept of secularism, i.e., there
Schedule Content is a complete separation of the State and
religion.
1. Second Emoluments, allowances,
Which of the statements given above is/are
privileges
not correct?
2. Fourth Allocation of seats in the
(a) 1 only
Rajya Sabha
(b) 3 only
3. Fifth Administration of tribal
(c) 1 and 3 only
areas in Assam,
Meghalaya, Tripura and (d) 1, 2 and 3
Mizoram
4. Sixth Administration and 4. With reference to the recent Supreme
control of scheduled Court verdict on ‘vulnerable witnesses’,
areas and scheduled consider the following statements:
tribes 1. Although ‘vulnerable witnesses’ include
Select the correct answer using the code victims of sexual assault under section 377
given below: of IPC, they exclude victims of any other
kind of sexual assault.
(a) Only one pair
2. The Parliament of India passed a
(b) Only two pairs
comprehensive witness protection scheme
(c) Only three pairs
in 2018.
(d) All four pairs
Which of the statements given above is/are
correct?
2. Consider the following statements: (a) 1 only
1. The scope of the power of judicial review (b) 2 only
of the Supreme Court in India is wider than (c) Both 1 and 2
that of what exists in the U.S.A.
(d) Neither 1 nor 2
2. The Indian parliamentary system is based
on the principle of strict separation of
powers between the legislative and 5. Consider the following statements
executive organs. regarding the features of the Regulating Act of
1773:
3. The Indian Constitution establishes an
independent judiciary unlike the US where 1. The British cabinet was given the right to
the judiciary is integrated. exercise control over East India Company’s
affairs in India for the first time.
Which of the statements given above is/are
not correct? 2. The Governor of Bengal was made the
Governor General of India, with an
(a) 3 only
executive council of four members to
(b) 2 and 3 only
assist him.
(c) 1 and 2 only
3. It excluded the matters arising in the
(d) 1, 2 and 3 collection of revenue from the jurisdiction
of the Supreme Court of Calcutta.
3. With reference to secularism in India, 4. It made the governors of Bombay and
consider the following statements: Madras presidencies subordinate to the
1. The word ‘Secularism’ was not present in Governor-general.
the original Constitution and was added Which of the statements given above are
through the 44th Constitutional correct?
Amendment Act. (a) 1, 2 and 3 only
2. The Supreme Court laid down that the
(b) 2, 3 and 4 only
Constitution is federal and held federalism
as part of its ‘basic feature’ in the S.R. (c) 1 and 4 only
Bommai case. (d) 1, 2, 3 and 4

1-A
SIP Polity TEST 01

6. With respect to the Pitt's India Act of 1784, down guidelines to be followed by
consider the following statements: establishments in dealing with complaints
1. It established a system of double about sexual harassment.
government in India. Which of the statements given above is/are
2. The Governor General was given an not correct?
effective casting vote in his executive (a) 1 only
council even if none of the members (b) 2 only
supported him. (c) Both 1 and 2
3. The Governor General was authorized to (d) Neither 1 nor 2
direct all military operations in India.
4. The possessions of the East India Company 10. Consider the following statements
in India came under the supremacy of the regarding the doctrine of ‘Rule of law’:
British Parliament.
1. It was propounded by Ivor Jennings.
Which of the statement given above is/are 2. Predominance of legal spirit constitutes
correct? one of its three aspects in India.
(a) 1 and 4 only Which of the statements given above is/are
(b) 1, 3 and 4 only correct?
(c) 2 and 3 only (a) 1 only
(d) 1, 2, 3 and 4 (b) 2 only
(c) Both 1 and 2
7. Which one of the following in Indian polity is (d) Neither 1 nor 2
an essential feature that indicates that it is
federal in character? 11. With reference to the Supreme Courts (SC)
(a) The independence of judiciary is of India and the United States of America,
safeguarded. consider the following statements:
(b) The Union Legislature has elected 1. The Indian SC has advisory jurisdiction
representatives from constituent units. while the American SC does not have any
(c) The Union Cabinet can have elected advisory jurisdiction.
representatives from regional parties. 2. The jurisdiction and powers of the Indian
(d) The Fundamental Rights enforceable by SC can be enlarged by the Parliament while
Courts of Law. the jurisdiction and powers of the
American SC are limited to that conferred
by the Constitution.
8. A law member was introduced into the
3. The Indian SC has power of judicial
executive council of Governor General
superintendence while the American SC
through which one of these Acts?
does not have any such power.
(a) The Charter Act of 1793 Which of the statements given above are
(b) The Charter Act of 1813 correct?
(c) The Charter Act of 1833 (a) 1 and 2 only
(d) The Charter Act of 1853 (b) 2 and 3 only
(c) 1 and 3 only
9. With reference to the prevention of sexual (d) 1, 2 and 3
harassment (POSH) trials, consider the
following statements: 12. Who among these described the Preamble
1. The recent guidelines laid down by the as the ‘horoscope of our sovereign democratic
Bombay High Court in this matter prohibit republic’?
the disclosure of the identities of the (a) K.M. Munshi
victim, accused and witnesses. (b) M.Hidayatullah
2. The Supreme Court passed a landmark (c) Sir Alladi Krishnaswami Iyer
judgment in the Shah Bano case laying (d) Sir Ernest Barker

2-A
SIP Polity TEST 01

13. How many of the following pairs is/are Which of the statement given above are
correctly matched? correct?
Scholar/Jur Criticism of the (a) 1 and 2 only
ist Indian Constitution (b) 1 and 3 only
1. N. Srinivasan ‘both in language and (c) 2 and 3 only
substance a close copy (d) 1, 2 and 3
of the GOI Act, 1935’
2. Ivor Jennings ‘many of its provisions 16. With reference to the British East India
are copied textually Company (EIC), consider the following
from the GOI Act, 1935’ statements:
3. H.V. Kamath ‘the bulkiest 1. The EIC was formed in 1599 by the British
Constitution that the government to trade with India and
world has produced’ Southeast Asia.
4. K. ‘the kind of 2. Queen Elizabeth I granted exclusive
Hanumantha Constitution Mahatma trading rights with India to the Company.
iya Gandhi did not 3. The EIC was dissolved by the Government
envisage’ of India Act of 1858.
Select the correct answer using the code Which of the statements given above is/are
given below: correct?
(a) Only one pair (a) 1 and 2 only
(b) Only two pairs (b) 2 only
(c) Only three pairs (c) 1 and 3 only
(d) All four pairs (d) 1, 2 and 3

14. With reference to Indian History, the 17. With reference to the Global Risks Report
Members of the Constituent Assembly from 2022, consider the following questions:
the Provinces were 1. The report was released by the World Bank
(a) Directly elected by the people of those in collaboration with World Economic
Provinces Forum.
(b) Nominated by the Indian National 2. The failure of technology governance and
Congress and the Muslim League digital inequality have been highlighted by
(c) Elected by the Provincial Legislative the report as major risks for the Indian
Assemblies economy.
3. The report highlights that the global
(d) Selected by the Government for their
economic recovery from the recession
expertise in constitutional matters
caused by COVID-19 pandemic has been
continuously improving due to integrated
15. Consider the following statements about supply chain system.
the Indian Councils Act of 1861: Which of the statements given above is/are
1. For the first time, it associated Indians with correct?
the Viceroy’s legislative council. (a) 1 and 2 only
2. It empowered the Viceroy to issue (b) 2 only
ordinances during an emergency.
(c) 1 and 3
3. It gave recognition to the portfolio system
(d) 1, 2 and 3
introduced by Lord Canning in 1859.

3-A
SIP Polity TEST 01

18. With reference to the ‘All-India Quarterly 21. Which of the following does not ensure
Establishment-based Employment Survey’ independence of the Judiciary in India?
(AQEES), consider the following statements: (a) Security of tenure of the judges
1. The Labor Bureau publishes it essentially (b) Fixed service conditions for the judges
to provide quarterly projections of (c) All the expenses of the Supreme Court
employment in eight core manufacturing charged on the Consolidated Fund of India
industrials in India.
(d) Judges of High Courts cannot be
2. The Area Frame Establishment Survey transferred against without their consent
under AQEES covers establishments
employing 9 or less workers.
22. With reference to the election expenditure
3. According to second round Quarterly
in India, consider the following statements:
Employment Survey, the overall
percentage of female workers has 1. According to the Constitution of India,
improved marginally compared to last every election candidate has to keep a
survey. separate account of all expenditure
incurred between the date of their
Which of the statements given above is/are
nomination and the date of declaration of
correct?
the result.
(a) 1 and 2 only
2. An incorrect account or expenditure
(b) 2 only beyond the cap can lead to disqualification
(c) 1 and 3 only of the candidate by the ECI for up to a
(d) 2 and 3 only period of seven years.
3. Though there is a cap on an individual
19. With reference to the Charter Act of 1833, election candidate’s expenditure, there is
which of these statements are correct? no cap on a political party’s expenditure.
1. It made the Governor-General of Bengal as Which of the statements given above is/are
the Governor-General of India. not correct?
2. It ended the commercial responsibility of (a) 1 only
the British East India Company in India. (b) 1 and 2 only
3. It introduced a system of open competition (c) 2 and 3 only
for selection of civil servants in India. (d) 1 and 3 only
Select the correct answer using the code given
below:
23. Which of the statements given below are
(a) 1 and 2 only correct with reference to Lok Adalat?
(b) 1 and 3 only 1. The first Lok Adalat camp in post-
(c) 2 and 3 only independence India was organized in
(d) 1, 2 and 3 Gujarat in 1982.
2. The institution of Lok Adalat has been
given statutory status under the Legal
20. Who among the following acclaimed that Services Authorities Act, 1987.
the Constitution of India has been framed after
3. No appeal shall lie to any court against the
‘ransacking all the known Constitutions of the
award of the Lok Adalat.
World’?
Select the correct answer from the codes given
(a) B.R. Ambedkar
below:
(b) Ivor Jennings
(a) 1 and 2 only
(c) N. Srinivasan
(b) 2 and 3 only
(d) P.R. Deshmukh
(c) 1 and 3 only
(d) 1, 2 and 3

4-A
SIP Polity TEST 01

24. Consider the following statements: 28. Which of the following constitute the
1. The Regulating Act of 1773 exempted the features of the parliamentary form of
Governor-General and the Council from government in India?
the jurisdiction of the Supreme Court. 1. Presence of both nominal and real
2. According to the Charter Act of 1793, the executives
members of the Board of Control were to 2. Parliamentary sovereignty
be paid out of the Indian revenues. 3. Collective responsibility of ministers to the
3. The Charter Act of 1813 ended the trade Parliament
monopoly of East India Company in India 4. Double Membership of the ministers
except in tea and trade with China. Select the correct answer using the code given
4. The Charter Act of 1833 empowered the below:
local governments in India to impose taxes (a) 1, 2 and 3 only
on persons. (b) 2, 3 and 4 only
Which of the statements given above are (c) 1, 3 and 4 only
correct? (d) 1, 2, 3 and 4
(a) 1, 2 and 4 only
(b) 2 and 3 only 29. With reference to Article 21, consider the
(c) 3 and 4 only following statements:
(d) 1, 2, 3 and 4 1. It contains the expression ‘due process of
law’, which is quite different from the
25. During the colonial rule, the legislative expression ‘procedure established by law’
council in India was given the power of contained in the American Constitution.
discussing the budget for the first time 2. In the Maneka Gandhi case, the Supreme
through which of these legislations? Court introduced the concept of
(a) Indian Councils Act of 1861 ‘procedure established by law’ through its
liberal interpretation.
(b) Indian Councils Act of 1892
Which of the statements given above is/are
(c) Indian Councils Act of 1909 not correct?
(d) Government of India Act of 1919 (a) 1 only
(b) 2 only
26. Which portfolio was held by Dr. Rajendra (c) Both 1 and 2
Prasad in the Interim Government formed in (d) Neither 1 nor 2
the year 1946?
(a) Defence 30. With reference to the Charter Act of 1853,
(b) External Affairs and Commonwealth consider the following statements:
(c) Food and Agriculture 1. It introduced local representation in the
(d) None Central Legislative Council.
2. The Committee on the Indian Civil Service
27. Which of the following Constitutional was appointed under the Chairmanship of
Amendment Acts gave constitutional status Lord Macaulay.
and protection to the co-operative societies? 3. All vacancies to the Indian Civil Services
(a) 42nd Constitutional Amendment Act were to be filled in by simultaneous exams
(b) 86th Constitutional Amendment Act to be held in India and England.
(c) 93rd Constitutional Amendment Act Which of the statements given above is/are
correct?
(d) 97th Constitutional Amendment Act
(a) 1 only
(b) 1 and 2 only
(c) 2 only
(d) 2 and 3 only

5-A
SIP Polity TEST 01

31. With reference to Morley-Minto reforms, 34. In which of the following cases did the
consider the following statements: Supreme Court first hold that the Indian
1. It separated the central and provincial Constitution is founded on the ‘bedrock of the
legislative subjects. balance between the Fundamental Rights and
2. It associated Indians with the executive the Directive Principles’?
council of the Viceroy for the first time. (a) Kesavananda Bharati v. State of Kerala
3. Bicameralism was introduced in the (b) Maneka Gandhi v. Union of India
central and provincial legislatures. (c) Minerva Mills v. Union of India
4. It provided for the establishment of a (d) ADM Jabalpur v. Shivakant Shukla
central public service commission.
5. The strength of the Central Legislative 35. With respect to languages used in the
Council was increased to 60 members. proceedings of various courts in India,
Which of the statements given above are consider the following statements:
correct? 1. The Constitution of India provides that all
(a) 1, 2, 3 and 4 only the proceedings in the Supreme Court and
(b) 3, 4 and 5 only in every High Court shall only be in English
language.
(c) 2 and 5 only
2. The President of India may authorize the
(d) 1, 2, 3, 4 and 5
use of Hindi or any other language used for
any official purpose of the state in the
32. With reference to Constituent Assembly, proceedings of the High Court of that
consider the following statements: state.
1. B.N. Rau put forward the idea of a 3. The state government has the power to
Constituent Assembly for India for the first declare any regional language as an
time. alternate language for the proceedings of
2. M.N. Roy was appointed as the legal subordinate courts.
advisor to the Constituent Assembly. Which of the statements given above is/are
Which of the statements given above is/are not correct?
correct? (a) 1 only
(a) 1 only (b) 1 and 2 only
(b) 2 only (c) 1 and 3 only
(c) Both 1 and 2 (d) 2 and 3 only
(d) Neither 1 nor 2
36. With reference to marital rape in India,
consider the following statements:
33. Constitutional government means
1. Section 375 of the IPC (Indian Penal Code)
(a) a representative government of a nation
exempts forceful sexual intercourse by a
with the federal structure
man with his own wife from the offence of
(b) a government whose Head enjoys nominal rape, provided the wife is above 18 years of
powers age.
(c) a government whose Head enjoys real 2. The Justice J. S. Verma Committee made
powers radical suggestions on criminalising
(d) a government limited by the terms of the marital rape.
Constitution Which of the statements given above is/are
correct?
(a) 1 only
(b) 2 only
(c) Both 1 and 2
(d) Neither 1 nor 2

6-A
SIP Polity TEST 01

37. With reference to the Government of India (a) 1 only


Act of 1858, consider the following
(b) 2 only
statements:
(c) Both 1 and 2
1. It substantially altered the system of
Government that prevailed in India. (d) Neither 1 nor 2
2. It promised equal opportunities to Indians
in government services irrespective of race 40. With reference to the Free Trade
or creed. Agreement, consider the following
3. It asserted the sovereignty of the British statements:
Crown over the East India Company’s 1. In India, there is no mechanism for any role
territories in India for the first time. of the Indian Parliament in the ratification
Which of the statements given above is/are of free trade agreements.
correct? 2. Free Trade Agreement is limited to only
(a) 1 and 2 only trade in goods and seeks only a tariff
elimination in terms of a margin of
(b) 2 only
preference.
(c) 1 and 3 only
3. Patent evergreening and generic
(d) 3 only medicines have been a contentious issue in
FTA negotiations between India and
United Kingdom.
38. Consider the following statements, with
respect to the Government of India Act of Which of the statements given above is/are
1935: not correct?
1. It provided for bicameralism in all the (a) 1 and 2 only
eleven provinces under the British (b) 2 only
administration. (c) 1 and 3
2. The residuary power of legislation was (d) 1, 2 and 3
vested with the federal government.
3. The provincial governments were allowed
to borrow money on their own security. 41. Which of these statements given below
is/are correct?
Which of the statements given above is/are
not correct? 1. The World Bank’s classification of the
world’s economies is based on estimates of
(a) 1 and 2 only gross national income per capita.
(b) 2 only 2. Recently, China got the ‘developed
(c) 2 and 3 only country’ status at the World Trade
Organisation due to rise in its per capita
(d) 3 only income.
3. The IMF provides status of ‘developing
39. Consider the following statements: country’ particularly on the basis of foreign
1. The Indian High Court Act of 1861 reserve exchange.
established the High Courts of Calcutta, Select the correct answer using the code
Bombay, and Madras. given below:
2. The Federal Court was set up in 1920 (a) 1 only
through the Government of India Act of (b) 1 and 2 only
1919.
(c) 1 and 3 only
Which of the statements given above is/are
correct? (d) 3 only

7-A
SIP Polity TEST 01

42. Which of the following statements with 3. Though the Indian Constitution is federal,
reference to the Montagu-Chelmsford it provides only for a single citizenship.
reforms are correct? Which of the statements given above is/are
1. It provided separate electorates for Anglo- correct?
Indians. (a) 3 only
2. The Act divided the powers between the (b) 1 and 3 only
Centre and provincial units in terms of (c) 1 and 2 only
three lists. (d) 1, 2 and 3
3. The provincial legislatures were allowed to
have their own separate budgets. 45. Consider the following statements:
4. It provided for dyarchy at provinces in 1. WTO has recently ruled that India’s sugar
which the reserved subjects were to be subsidy policy is against the General
administered by the governor with the aid Agreement on Tariffs and Trade.
of ministers.
2. According to WTO rules, subsidies
Select the correct option using the code given provided to sugarcane cultivators cannot
below: exceed 10% of the total value of sugar
(a) 1, 2 and 4 only production.
(b) 1 and 3 only 3. The judgments once passed by the WTO’s
(c) 2, and 4 only Appellate Body immediately become final
(d) 1, 2, 3 and 4 and binding on the signatory members of
WTO.
43. With reference to the Goods and Services Which of the statements given above is/are
Tax, consider the following statements: correct?
1. The Centre has released the entire amount (a) 1 and 2 only
of GST compensation payable to States up (b) 1 and 3 only
to 31st May, 2022. (c) 2 and 3 only
2. The Compensation Fund, which was (d) 1, 2 and 3
financed by a Cess imposed on all goods
and services subject to GST, was used to 46. Consider the following statements:
compensate the states. 1. The Russian invasion of Ukraine is the
3. The GST compensation Cess collection has largest conventional conflict that Europe
been extended till March 2026 but only to has seen since World War II.
repay the loans taken by the Union 2. For the first time, a NATO-Russia council
government to compensate States GST was set up in 2022 to discuss geopolitical
revenue loss. issues.
Which of the statements given above is/are 3. Minsk Agreement particularly aims at
correct? finding permanent peaceful solution to the
(a) 1 only geopolitical tension between NATO
(b) 1 and 2 only countries and Russia.
(c) 1 and 3 only Which of the statements given above is/are
(d) 3 only correct?
(a) 1 only
44. Consider the following statements: (b) 1 and 2
1. At the time of Indian independence, the (c) 2 only
universal adult suffrage was granted to all (d) 1 and 3
the adult citizens aged 21 years and above.
2. The voting age was reduced to 18 years
from 21 years in 1985 by the 53rd
Constitutional Amendment Act.

8-A
SIP Polity TEST 01

47. Consider the following statements: 49. Who among these were the members of
1. India was declared an independent and the Drafting Committee of the Constituent
sovereign state on August 15, 1947 through Assembly?
the Indian Independence Act, 1947. 1. Dr. B.R. Ambedkar
2. The Nehru report of 1928 provided 2. Pattabhi Sitaramayya
fundamental rights and universal adult 3. N. Gopalaswamy Ayyangar
suffrage. 4. Alladi Krishnaswami Ayyar
3. The office of the Secretary of State for Select the correct answer using the code
India was abolished by the Indian given below:
Independence Act, 1947. (a) 1 and 2 only
Which of the statements given above is/are (b) 3 and 4 only
correct? (c) 1, 3 and 4 only
(a) 1 only (d) 1, 2, 3 and 4
(b) 1 and 2 only
(c) 2 and 3 only 50. Which of the following statements is not
(d) 3 only correct?
(a) The Constitution was adopted on 26
48. Consider the following statements: November, 1949.
1. Assam was carved out of Meghalaya under (b) The date of commencement of
the Meghalaya Reorganisation Act, 1971. Constitution was chosen as 26 January
2. Langpih was part of the Garo Hills during because Purna Swaraj day was celebrated
the British colonial period but post- on this date in 1930.
Independence, it became part of the Mikir (c) The Indian Independence Act of 1947 and
hills in Assam. the Government of India Act of 1935 were
Which of the statements given above is/are repealed with the commencement of the
Constitution.
not correct?
(d) The adopted Constitution contained
(a) 1 only
Preamble, 395 Articles and ten Schedules.
(b) 2 only
(c) Both 1 and 2
(d) Neither 1 nor 2

9-A
SIP POLITY TEST 01

SIP POLITY TEST #1


ANSWER KEY

S. No. Answer S. No. Answer S. No. Answer S. No. Answer S. No. Answer
1. B 11. D 21. D 31. C 41. A
2. D 12 A 22. B 32. D 42. B
3. C 13. D 23. D 33. D 43. C
4. D 14. C 24. B 34. C 44. B
5. C 15. D 25. B 35. B 45. A
6. A 16. B 26. C 36. B 46. A
7. A 17. B 27. D 37. B 47. C
8. C 18. D 28. C 38 A 48. C
9. B 19. A 29. C 39. A 49. C
10. D 20. A 30. B 40. B 50. D

For More Study Material, Visit: studyiq.com Page no.


1
SIP POLITY TEST 01

Explanation
1. Answer: B
Explanation:
● Pair 1 is correctly matched: Second Schedule – Provisions relating to the emoluments,
allowances, privileges and so on of:
o The President of India
o The Governors of States
o The Speaker and the Deputy Speaker of the Lok Sabha
o Etc.
● Pair 2 is correctly matched: Fourth Schedule – Allocation of seats in the Rajya Sabha to the states
and the union territories.
● Pair 3 is incorrectly matched: Fifth Schedule – Provisions relating to the administration and
control of scheduled areas and scheduled tribes.
● Pair 4 is incorrectly matched: Sixth Schedule – Provisions relating to the administration of tribal
areas in the states of Assam, Meghalaya, Tripura and Mizoram.
Question Hack: Schedule 5 is a more general Schedule providing provisions regarding Scheduled areas
and Scheduled tribes. Schedule 6 is more particular as it contains provisions regarding tribal areas in
more vulnerable areas of four states. Hence, it is obvious that specific provisions would follow and
come after the general provisions.
Source: Indian Polity by M Laxmikanth 6th edition, Chapter 3

2. Answer: D
Explanation:
● Statement 1 is incorrect: The scope of judicial review power of the Supreme Court in India is
narrower than that of what exists in the USA. This is because the American Constitution provides
for ‘due process of law’ against that of ‘procedure established by law’ contained in the Indian
Constitution (Article 21). The framers of the Indian Constitution have preferred a proper synthesis
between the British principle of parliamentary sovereignty and the American principle of judicial
supremacy.
● Statement 2 is incorrect: The Constitution of India has opted for the British Parliamentary System
of Government rather than the American Presidential System of Government. The parliamentary
system is based on the principle of co-operation and co-ordination between the legislative and
executive organs while the presidential system is based on the doctrine of separation of powers
between the two organs.
● Statement 3 is incorrect: The Indian Constitution establishes a judicial system that is integrated
as well as independent. The Supreme Court stands at the top of the integrated judicial system
in the country. Below it, there are high courts at the state level. Under a high court, there is a
hierarchy of subordinate courts, that is, district courts and other lower courts. This single system
of courts enforces both the central laws as well as the state laws, unlike in the USA having an
independent judiciary, where the federal laws are enforced by the federal judiciary and the
state laws are enforced by the state judiciary.
Question Hack: In India, all ministers including the Prime Minister are members of Parliament. So,
how can we have strict separation of powers between the executive and the legislature!
Source: Indian Polity by M Laxmikanth 6th edition, Chapter 3

3. Answer: C
Explanation:
● Statement 1 is incorrect: The Preamble has been amended only once so far, in 1976, by the 42nd
Constitutional Amendment Act, which has added three new words – Socialist, Secular and
Integrity–to the Preamble. This amendment was held to be valid.

For More Study Material, Visit: studyiq.com Page no.


1
SIP POLITY TEST 01

● Statement 2 is correct: In S.R. Bommai case (1994), the Supreme Court laid down that the
Constitution is federal and characterized federalism as its ‘basic feature’. It observed: “The fact
that under the scheme of our Constitution, greater power is conferred upon the Centre vis-a-vis
the states does not mean that the states are mere appendages of the Centre. The states have an
independent constitutional existence. They are not satellites or agents of the Centre. Within the
sphere allotted to them, the states are supreme. The fact that during emergency and in certain
other eventualities their powers are overridden or invaded by the Centre is not destructive of the
essential federal feature of the Constitution. They are exceptions and the exceptions are not a
rule. Let it be said that the federalism in the Indian Constitution is not a matter of administrative
convenience, but one of principle–the outcome of our own process and a recognition of the
ground realities”.
● Statement 3 is incorrect: The Western concept of secularism connotes a complete separation
between the religion (the church) and the state (the politics). This negative concept of secularism
is inapplicable in the Indian situation where the society is multireligious. Hence, the Indian
Constitution embodies the positive concept of secularism, i.e., giving equal respect to all
religions or protecting all religions equally.
Source: Indian Polity by M Laxmikanth 6th edition, Chapters 3, 4 and 26

4. Answer: D
Explanation:
The Supreme Court (SC) of India recently expanded the meaning of vulnerable witnesses to make it
more inclusive and as a result, include among others sexual assault victims, those with mental illness
and people with speech or hearing impairment.
● Statement 1 is incorrect: Vulnerable witnesses will now include not only child witnesses, but
will also include –
o Age-neutral victims of sexual assault.
o Gender-neutral victims of sexual assault, under section 377 IPC (unnatural offences).
o Witnesses suffering from mental illness as defined in Mental Healthcare Act, 2017.
o Witnesses with threat perception and any speech or hearing impaired individual or person
suffering from any other disability.
● Statement 2 is incorrect: In 2018, the SC approved the Witness Protection Scheme 2018 which
aimed at enabling a witness to depose fearlessly and truthfully. Under the Judgement, SC held
that:
o Right of witnesses to testify freely in courts is part of Article 21 (Right to Life).
o The scheme will be the law under Article 141/142 of the Constitution of India.
o The bench has also asked all States and UTs to set up vulnerable witness deposition
complexes.
While the scheme is pending in the Parliament, the SC had ordered to implement the scheme
immediately in all the states and the scheme would be the law of the land. The need to protect
witnesses has been emphasized by Law Commission reports and court judgments for years. The State
of Gujarat v. Anirudh Singh (1997), 14th Law Commission Report and Malimath Committee Report
also have recommended a witness protection scheme.
Source:
https://main.sci.gov.in/supremecourt/2019/32085/32085_2019_34_1_32533_Judgement_11-Jan-
2022.pdf

5. Answer: C
Explanation:
● Statement 1 is correct: The Regulating Act, 1773 brought about the British government’s
involvement in Indian affairs in the effort to control and regulate the functioning of the East India
Company. It recognized that the Company’s role in India extended beyond mere trade to

For More Study Material, Visit: studyiq.com Page no.


2
SIP POLITY TEST 01

administrative and political fields, and introduced the element of centralized administration. The
directors of the Company were required to submit all correspondence regarding revenue affairs
and civil and military administration to the government. Thus, for the first time, the British
cabinet was given the right to exercise control over Indian affairs.
● Statement 2 is incorrect: It designated the Governor of Bengal as the ‘Governor-General of
Bengal’ (not Governor General of India as that was done through the Charter Act of 1833) and
created an Executive Council of four members to assist him. It was to oversee the civil and
military aspects of government. They were required to function according to the majority rule.
The first such Governor General was Lord Warren Hastings.
● Statement 3 is incorrect: The Regulating Act of 1773 provided for the establishment of a Supreme
Court at Calcutta (1774) comprising one chief justice and three other judges. The Amending Act
of 1781, also known as the Act of Settlement excluded the revenue matters and the matters
arising in the collection of revenue from the jurisdiction of the Supreme Court. It exempted the
Governor-General and the Council from the jurisdiction of the Supreme Court for the acts done
by them in their official capacity. Similarly, it also exempted the servants of the company from the
jurisdiction of the Supreme Court for their official actions.
● Statement 4 is correct: It made the governors of Bombay and Madras presidencies subordinate
to the Governor-General of Bengal, unlike earlier, when the three presidencies were independent
of one another. The governor-general could exercise some powers over Bombay and Madras, a
vague provision which created many problems. The whole scheme was based on checks and
balances.
Question Hack: William Bentinck was the first Governor General of India and not Warren Hastings.
So, it couldn’t have been Regulating Act of 1773.
Sources:
Indian Polity by M. Laxmikanth, 6th Edition, Chapter 1- Historical Background
Spectrum’s Brief history of Modern by Rajiv Ahir, Chapter 26- Constitutional, Administrative and
Judicial Developments.

6. Answer: A
Explanation:
● Statement 1 is correct: The Pitt’s India Act 1784 distinguished between the commercial and
political functions of the Company. It allowed the Court of Directors to manage the commercial
affairs, but created a new body called Board of Control to manage the political affairs. Thus, it
established a system of double government.
● Statement 2 is incorrect: The Constitution of the Company’s government in India was revised. The
Act established the principle that the government of India be placed under the Governor
General and a Council of three, so that if only one member of the Council supported him, he
could have his way. The Governor General was given a casting vote. The Act clearly stated that
the Presidencies of Madras and Bombay were to be subordinate to the Presidency of Bengal in all
matters of war, diplomatic relations and revenue.
● Statement 3 is incorrect: The Pitt’s India Act 1784 established a Board of Control consisting of six
commissioners, including, two Cabinet ministers. The Board of Control was to guide and control
the work of the Court of Directors and the Government of India. They were to control all matters
of civil and military Government of the British territories in India. A secret committee consisting
of three Directors was appointed to take the place of the Court of Directors in political and military
matters. The Governor General and council were made subordinate to the British Government.
They were forbidden to declare war and enter into any treaty without the sanction of the
directors or the secret committee.
● Statement 4 is correct: The possessions of the Company in India came under the supremacy of
the British Parliament. The Act laid the foundation of a centralized administration-a process which
reached its climax towards the close of the nineteenth century. Parliament's control over the East

For More Study Material, Visit: studyiq.com Page no.


3
SIP POLITY TEST 01

lndia Company was tightened, a trend which remained conspicuous till the Crown directly took
over the Government of lndia in 1858.
Source:
https://egyankosh.ac.in/bitstream/123456789/20342/1/Unit-23.pdf

7. Answer: A
Explanation:
● Option (a) is correct: A federal system is different from a unitary system in that sovereignty is
constitutionally split between two territorial levels so that each level can act independently of the
others in some areas. The establishment of an independent judiciary is inextricably linked to the
federal nature of Indian polity. The Constitution establishes an independent judiciary, led by the
Supreme Court, to resolve disputes between the Centre and the states.
Source: UPSC Civil Services Examination (Prelims) 2021

8. Answer: C
Explanation:
● Option (c) is correct: By the Act of 1833, the Governor General in Council were given the power
to legislate for the whole of the British territories in India. These laws were applicable to all
persons, British or Indian foreigners or others and to the servants of the Company. They were
enforceable by all courts in India. The Act added one more member to the Executive council of
the Governor General, the Law Member, whose work was fully legislative. He had no vote in the
Council and he was to attend meetings, on invitation. But he practically became a regular member
of the council. Lord Macaulay, the Law member, influenced the educational policy of the
government for a number of years.
Question Hack: Lord T.B Macaulay was the first law member of the Governor General’s executive
council which is a well-known fact. He presented his famous minutes on education in the year 1835,
which makes the 1833 Act the most plausible answer.
Source:
https://egyankosh.ac.in/bitstream/123456789/20342/1/Unit-23.pdf

9. Answer: B
Explanation:
● Statement 1 is correct: In a bid to protect the identities of those involved in POSH (prevention of
sexual harassment) trials, the Bombay High Court, in a recent judgment, has set out certain
guidelines. The recent guidelines laid down by Justice Gautam Patel prohibit the disclosure of
the identities of the victim, accused and witnesses, and mandate that all such court hearings be
held in-camera or in Judges’ Chambers, with only the court stenographer, plaintiff, defendant
and their lawyers being present. Even court orders and judgments will not be delivered in open
court. Parties to POSH trials are prohibited from disclosing any information relating to such trials
(including the final order/judgment) to the media or publicizing the same via social media, without
securing permission from the Court.
● Statement 2 is incorrect: In 1997, the Supreme Court passed a landmark judgment in the Vishaka
case (not Shah Bano case which was about maintenance for Muslim women after divorce) laying
down guidelines to be followed by establishments in dealing with complaints about sexual
harassment. Vishaka Guidelines were stipulated by the Supreme Court of India, in Vishakha and
others v State of Rajasthan case in 1997, regarding sexual harassment at workplace. The court
stated that these guidelines were to be implemented until legislation is passed to deal with the
issue. They were superseded in 2013 by the Sexual Harassment of Women at Workplace
(Prevention, Prohibition and Redressal) Act.
Sources:
http://gptsoraba.in/english/doc/vishakaguidelines.pdf

For More Study Material, Visit: studyiq.com Page no.


4
SIP POLITY TEST 01

https://indianexpress.com/article/opinion/columns/bombay-hc-posh-guidelines-sexual-harassment-
7570713/
10. Answer: D
Explanation:
● Statement 1 is incorrect: The concept of ‘Rule of Law’ was propounded and made famous by
A.V. Dicey, a British jurist. His concept has the following three elements or aspects:
o Supremacy of the law – Absence of arbitrary power, that is, no man can be punished except
for a breach of law.
o Equality before the law – Equal subjection of all citizens (rich or poor, high or low, official or
non-official) to the ordinary law of the land administered by the ordinary law courts.
o Predominance of Legal Spirit – The primacy of the rights of the individual, that is, the
constitution is the result of the rights of the individual as defined and enforced by the courts
of law rather than the constitution being the source of the individual rights.
● Statement 2 is incorrect: The first and the second elements are applicable to the Indian System
and not the third one. In the Indian System, the constitution is the source of the individual rights.
Thus, there is no predominance of legal spirit in India. The Supreme Court held that the ‘Rule of
Law’ as embodied in Article 14 is a ‘basic feature’ of the constitution. Hence, it cannot be
destroyed even by an amendment.
Source: Indian Polity by M Laxmikanth 6th edition, Chapter 7

11. Answer: D
Explanation:
Option (d) is correct:
Indian Supreme Court American Supreme Court

1. Its original jurisdiction is confined to federal 1. Its original jurisdiction covers not only
cases. federal cases but also cases relating to naval
2. Its appellate jurisdiction covers forces, maritime activities, ambassadors,
constitutional, civil and criminal cases. etc.
3. It has a very wide discretion to grant special 2. Its appellate jurisdiction is confined to
leave to appeal in any matter against the constitutional cases only.
judgement of any court or tribunal (except 3. It has no such plenary power.
military). 4. It has no advisory jurisdiction.
4. It has advisory jurisdiction. 5. Its scope of judicial review is very wide.
5. Its scope of judicial review is limited. 6. It defends the rights of the citizen according
6. It defends the rights of the citizen according to the ‘due process of law’.
to the ‘procedure established by law’. 7. Its jurisdiction and powers are limited to
7. Its jurisdiction and powers can be enlarged that conferred by the Constitution.
by Parliament. 8. It has no such power due to the double (or
8. It has power of judicial superintendence separated) judicial system.
and control over state high courts due to an
integrated judicial system.
Source: Indian Polity by M Laxmikanth 6th edition, Chapter 26

12. Answer: A
Explanation:
● Option (b) is correct: According to K.M. Munshi, a member of the Drafting Committee of the
Constituent Assembly, the Preamble is the ‘horoscope of our sovereign democratic republic’.
● Option (b) is incorrect: M. Hidayatullah, a former Chief Justice of India, observed, ‘Preamble
resembles the Declaration of Independence of the United States of America, but is more than a

For More Study Material, Visit: studyiq.com Page no.


5
SIP POLITY TEST 01

declaration. It is the soul of our Constitution, which lays down the pattern of our political society.
It contains a solemn resolve, which nothing but a revolution can alter’.
● Option (c) is incorrect: In the words of Sir Alladi Krishnaswami Iyer, a member of the Constituent
Assembly who played a significant role in making the Constitution, ‘The Preamble to our
Constitution expresses what we had thought or dreamt so long’.
● Option (d) is incorrect: Sir Ernest Barker, a distinguished English political scientist, described the
Preamble as the ‘key-note’ to the Constitution. He said that the Preamble of the Indian
Constitution states ‘in a brief and pithy form the argument of much of the book; and it may
accordingly serve as a key-note’.
Source: Indian Polity by M.Laxmikanth, 6th edition, Chapter-4-Preamble of the Constitution.

13. Answer: D
Explanation:
● Pairs 1 and 2 are correctly matched: The critics said that the framers of the Constitution have
included a large number of the provisions of the Government of India Act of 1935 into the
Constitution of India. Hence, they called the Constitution as a “Carbon Copy of the 1935 Act” or
an “Amended Version of the 1935 Act”. For example, N. Srinivasan observed that the Indian
Constitution is “both in language and substance a close copy of the Act of 1935”. Similarly, Sir
Ivor Jennings, a British Constitutionalist, said that “the Constitution derives directly from the
Government of India Act of 1935 from which, in fact, many of its provisions are copied almost
textually”.
● Pair 3 is correctly matched: The critics stated that the Indian Constitution is too bulky and too
detailed and contains some unnecessary elements. Sir Ivor Jennings, a British Constitutionalist,
observed that the provisions borrowed were not always well-selected and that the constitution,
generally speaking, was too long and complicated. In this context, H.V. Kamath, a member of the
Constituent Assembly, commented: “The emblem and the crest that we have selected for our
assembly is an elephant. It is perhaps in consonance with that our constitution too is the bulkiest
that the world has produced”. He also said: “I am sure, the House does not agree that we should
make the Constitution an elephantine one”.
● Pair 4 is correctly matched: According to the critics, the Indian Constitution is un-Gandhian
because it does not contain the philosophy and ideals of Mahatma Gandhi, the father of the Indian
Nation. They opined that the Constitution should have been raised and built upon village
panchayats and district panchayats. In this context, the same member of the Constituent
Assembly, K. Hanumanthaiya, said: “That is exactly the kind of Constitution Mahatma Gandhi
did not want and did not envisage”.
Source: Indian Polity by M Laxmikanth 6th edition, Chapter 3

14. Answer: C
Explanation:
Option (c) is correct: The Constituent Assembly was constituted in November 1946 under the scheme
formulated by the Cabinet Mission Plan. The total strength of the Constituent Assembly was to be 389.
Of these, 296 seats were to be allotted to British India and 93 seats to the princely states. The
constituent assembly was partly nominated and partly elected.
● The members from the provinces were to be indirectly elected by the members of the provincial
assemblies, who themselves were elected on a limited franchise and;
● The representatives of the princely states were to be nominated by the heads of the princely
states.
Source: UPSC Civil Services Examination (Prelims) 2013

15. Answer: D
Explanation:

For More Study Material, Visit: studyiq.com Page no.


6
SIP POLITY TEST 01

● Statement 1 is correct: Indian Councils Act of 1861 made a beginning of the representative
institutions by associating Indians with the law-making process. It, thus, provided that the
Viceroy should nominate some Indians as non-official members of his expanded council. In 1862,
Lord Canning, the then Viceroy, nominated three Indians to his legislative council–the Raja of
Benaras, the Maharaja of Patiala and Sir Dinkar Rao.
● Statement 2 and 3 are correct: The Indian Councils Act of 1861 empowered the Viceroy to make
rules and orders for the more convenient transaction of business in the council. It also gave
recognition to the ‘portfolio’ system, introduced by Lord Canning in 1859. Under this, a member
of the Viceroy’s council was made in-charge of one or more departments of the Government and
was authorised to issue final orders on behalf of the council on matters of his department(s).
Moreover, it empowered the Viceroy to issue ordinances, without the concurrence of the
legislative council, during an emergency. The life of such an ordinance was six months.
Source: Indian Polity by M. Laxmikanth, 6th Edition, Chapter 1- Historical Background

16. Answer: B
Explanation:
● Statement 1 is incorrect: In 1599, a group of English merchants calling themselves the
‘Merchant Adventurers’ formed the company. Francis Drake’s voyage around the world in 1580
and the English victory over the Spanish Armada in 1588 generated a new sense of enterprise in
the British, encouraging sailors to venture out to the East. As the knowledge grew of the high
profits earned by the Portuguese in Eastern trade, English traders too wanted a share.
● Statement 2 is correct: On December 31, 1600, Queen Elizabeth I issued a charter with rights of
exclusive trading to the company named the ‘Governor and Company of Merchants of London
trading into the East Indies’. Initially, a monopoly of fifteen years was granted, which in May 1609
was extended indefinitely by a fresh charter. As the Dutch were already concentrating more on
the East Indies, the English turned to India in search of textiles and other commodities of trade.
● Statement 3 is incorrect: A number of things contributed to the end of the East India Company.
It acquired control of Bengal on the Indian subcontinent in 1757, and, as the company was an
agent of British imperialism, its shareholders were able to influence British policy there. This
eventually led to government intervention. The Regulating Act (1773) and the India Act (1784)
established government control of political policy. The company’s commercial monopoly was
broken in 1813, and from 1834 it was merely a managing agency for the British government of
India. It lost that role after the Indian Mutiny (1857). In 1873 it ceased to exist as a legal entity. It
was formally dissolved in 1874 by the East India Stock Dividend Redemption Act (1873).
Question Hack: East India Company was a private company so it couldn’t have been
formed/established by the British Government.
Sources: Brief history of Modern India Spectrum, Chapter 3 Advent of the Europeans in India
https://www.britannica.com/topic/East-India-Company

17. Answer: B
Explanation:
● Statement 1 is incorrect: The Global Risks Report 2022 was released by the World Economic
Forum (WEF) in January 2022. The Global Risks Report 2022 has been developed with the support
of the WEF's Global Risks Advisory Board, besides from ongoing collaboration with its strategic
partners, Marsh McLennan, SK Group and Zurich Insurance Group, and its academic advisers at
the Oxford Martin School (University of Oxford), the National University of Singapore and the
Wharton Risk Management and Decision Processes Center (University of Pennsylvania).
● Statement 2 is correct: The Global Risks Report 2022 presents the results of the latest Global Risks
Perception Survey (GRPS), followed by an analysis of key risks emanating from current economic,
societal, environmental and technological tensions. With regard to India, the report said fracture
of interstate relations, debt crises in large economies, widespread youth disillusionment, failure

For More Study Material, Visit: studyiq.com Page no.


7
SIP POLITY TEST 01

of technology governance and digital inequality are the top five risks identified for the Indian
economy.
● Statement 3 is incorrect: As per the report, the global economic recovery from the recession
caused by responses to the COVID-19 pandemic continues but is slowing. After a contraction of
3.1% in 2020, global economic growth is expected to reach 5.9% in 2021 and slow to 4.9% in 2022.
By 2024, the global economy is projected to be 2.3% smaller than it would have been without the
pandemic. The economic disruption from the pandemic has also created stronger incentives in
the vaccinated world to prioritize resilience over cost minimization. Governments and industries
may now drive regional convergence at the expense of global integration as they seek to
minimize supply chain disruptions.
● Additional Information:
Some major reports published by WEF are:
o Energy Transition Index.
o Global Competitiveness Report.
o Global IT Report- WEF along with INSEAD, and Cornell University publishes this report.
o Global Gender Gap Report.
o Global Risk Report.
o Global Travel and Tourism Report.
Sources:
https://www.thehindu.com/sci-tech/technology/rising-digital-dependence-during-pandemic-
heightening-cyber-threatssurvey-says/article38238916.ece
https://www.weforum.org/reports/global-risks-report-2022/digest

18. Answer: D
Explanation:
● Statement 1 is incorrect: The All-India Quarterly Establishment-based Employment Survey
(AQEES) has been taken up by the Labour Bureau to provide quarterly estimates about the
employment and related variables of establishments in both organized and unorganized
segments of nine selected sectors which account for a majority of the total employment in the
non-farm establishments. The nine selected sectors are Manufacturing, Construction, Trade,
Transport, Education, Health, and Accommodation& Restaurant, IT / BPO and Financial Services.
● Statement 2 is correct: All India Quarterly Establishment based Employment Survey (AQEES)”
has two components - first, the “Quarterly Employment Survey (QES)”and the second “Area
Frame Establishment Survey (AFES)”. The former relates to the establishments employing 10 or
more workers whereas the later relates to the establishments employing 9 or less workers.
● Statement 3 is correct: The estimated total employment in the nine selected sectors in second
round Quarterly Employment Survey (July-September, 2021) came out as 3.10 crore
approximately, which is 2 lakhs higher than the estimated employment (3.08 crore) from the first
round of QES (April-June, 2021). It is evident that the over-all percentage of female workers in
second round QES stood at 32.1%, which is higher than 29.3% reported during the first round
of QES.
Sources:
https://pib.gov.in/PressReleasePage.aspx?PRID=1788945
https://pib.gov.in/PressReleaseIframePage.aspx?PRID=1862597

19. Answer: A
Explanation:
Option (a) is correct: The Charter Act of 1833was the final step towards centralization in British India.
● Statement 1 is correct: It made the Governor-General of Bengal as the Governor-General of India
and vested in him all civil and military powers. Thus, the act created, for the first time,
Government of India having authority over the entire territorial area possessed by the British in

For More Study Material, Visit: studyiq.com Page no.


8
SIP POLITY TEST 01

India. Lord William Bentick was the first Governor-General of India. Furthermore, it deprived the
Governor of Bombay and Madras of their legislative powers. The Governor-General of India was
given exclusive legislative powers for the entire British India.
● Statement 2 is correct: It ended the activities of the East India Company as a commercial body,
which became a purely administrative body. It provided that the Company’s territories in India
were held by it ‘in trust for His Majesty, His heirs and successors’.
● Statement 3 is incorrect: It made an attempted to introduce a system of open competition for
selection of civil servants and stated that the Indians should not be debarred from holding
anyplace, office and employment under the Company. However, this provision was negated after
opposition from the Court of Directors.
Source:
Indian Polity by M. Laxmikanth, 6th Edition, Chapter – 1

20. Answer: A
Explanation:
● Option (a) is correct: The Constitution of India has borrowed most of its provisions from the
Constitutions of various other countries as well as from the Government of India Act of 1935. Dr.
B.R. Ambedkar proudly acclaimed that the Constitution of India has been framed after
‘ransacking all the known Constitutions of the World’. The structural part of the Constitution is,
to a large extent, derived from the Government of India Act of 1935. The philosophical part of the
Constitution (the Fundamental Rights and the Directive Principles of State Policy) derive their
inspiration from the American and Irish Constitutions, respectively. The political part of the
Constitution (the principle of Cabinet Government and the relations between the Executive and
the Legislature) have been largely drawn from the British Constitution. The other provisions of the
Constitution have been drawn from the Constitutions of Canada, Australia, Germany, USSR (now
Russia), France, South Africa, Japan and so on.
Source: Indian Polity by M Laxmikanth 6th edition, Chapter 3

21. Answer: D
Explanation:
● Options (a), (b) and (c) are correct: The Supreme Court is a federal court, the highest court of
appeal, the guarantor of the fundamental rights of the citizens and the guardian of the
Constitution. Hence, the Constitution has made various provisions to ensure its independence–
security of tenure of the judges, fixed service conditions for the judges, all the expenses of the
Supreme Court charged on the Consolidated Fund of India, prohibition on discussion on the
conduct of judges in the legislatures, ban on practice after retirement, power to punish for its
contempt vested in the Supreme Court, separation of the judiciary from the executive, and so on.
● Option (d) is incorrect: The President can transfer a judge from one high court to another after
consulting the Chief Justice of India. On transfer, he is entitled to receive in addition to his salary
such compensatory allowance as may be determined by Parliament. In 1977, the Supreme Court
ruled that the transfer of high court judges could be resorted to only as an exceptional measure
and only in public interest and not by way of punishment. Again in 1994, the Supreme Court held
that judicial review is necessary to check arbitrariness in transfer of judges. But, only the judge
who is transferred can challenge it. In the Third Judges case (1998), the Supreme Court opined
that in case of the transfer of high court judges, the Chief Justice of India should consult, in
addition to the collegium of four senior-most judges of the Supreme Court, the chief justice of
the two high courts (one from which the judge is being transferred and the other receiving him).
Thus, the sole opinion of the chief justice of India does not constitute the ‘consultation’ process.
Source: Indian Polity by M Laxmikanth 6th edition, Chapters 3 and 34

22. Answer: B

For More Study Material, Visit: studyiq.com Page no.


9
SIP POLITY TEST 01

Explanation:
Recently, the expenditure limit for candidates for Lok Sabha constituencies was increased from Rs 54
lakh-Rs 70 lakh (depending on states) to Rs 70 lakh-Rs 95 lakh, by the Election Commission of India
(ECI).
● Statement 1 is incorrect: Election Expenditure Limit is the amount an election candidate can
legally spend for their election campaign and has to account for, which includes expenses on
public meetings, rallies, advertisements, posters, banners, vehicles and advertisements. Under
Section 77 of the Representation of the People Act (RPA), 1951 (not the Constitution), every
candidate shall keep a separate and correct account of all expenditure incurred between the
date on which they have been nominated and the date of declaration of the result.
● Statement 2 is incorrect: All candidates are required to submit their expenditure statement to the
ECI within 30 days of the completion of the elections. An incorrect account or expenditure beyond
the cap can lead to disqualification of the candidate by the ECI for up to three years, under
Section 10A of RPA, 1951. The limit prescribed by the ECI is meant for legitimate expenditure
because a lot of money in elections is spent for illegitimate purposes. It has often been argued
that these limits are unrealistic as the actual expenditure incurred by the candidate is much higher.
● Statement 3 is correct: In December 2019, a private member’s bill was introduced in the
Parliament which intended to do away with the cap on election spending by candidates. The move
was taken on the grounds that the ceiling on election expenses ends up being counterproductive
by encouraging candidates to under-report their expenditure. There is no cap on a political
party’s expenditure, which is often exploited by candidates of the party. However, all registered
political parties have to submit a statement of their election expenditure to the ECI within 90 days
of the completion of the elections.
Sources:
https://indiankanoon.org/doc/117675817/
https://eci.gov.in/files/file/13928-limits-of-candidate%E2%80%99s-expenses-enhanced/

23. Answer: D
Explanation:
● Statements 1 and 2 are correct: The first Lok Adalat camp in the post-independence era was
organized in Gujarat in 1982. This initiative proved very successful in the settlement of disputes.
Consequently, the institution of Lok Adalat started spreading to other parts of the country. At that
time, this institution was functioning as a voluntary and conciliatory agency without any statutory
backing for its decisions. In view of its growing popularity, there arose a demand for providing a
statutory backing to this institution and the awards given by Lok Adalats. Hence, the institution
of Lok Adalat has been given statutory status under the Legal Services Authorities Act, 1987.
● Statement 3 is correct: An award of a Lok Adalat shall be deemed to be a decree of a Civil Court
or an order of any other court. Every award made by a Lok Adalat shall be final and binding on all
the parties to the dispute. No appeal shall lie to any court against the award of the Lok Adalat.
Source: https://nalsa.gov.in/lok-adalat

24. Answer: B
Explanation:
● Statement 1 is incorrect: The Amending Act of 1781, also known as the Act of Settlement
exempted the Governor-General and the Council from the jurisdiction of the Supreme Court for
the acts done by them in their official capacity. Similarly, it also exempted the servants of the
company from the jurisdiction of the Supreme Court for their official actions. It excluded the
revenue matters and the matters arising in the collection of revenue from the jurisdiction of the
Supreme Court. It provided that the Supreme Court was to have jurisdiction over all the
inhabitants of Calcutta.

For More Study Material, Visit: studyiq.com Page no.


10
SIP POLITY TEST 01

● Statement 2 is correct: The Charter Act of 1793 laid down that the members of the Board of
Control and their staff were to be paid out of the Indian revenues.
● Statement 3 is correct but statement 4 is incorrect: The Charter Act of 1813 abolished the trade
monopoly of the company in India that is, the Indian trade was thrown open to all British
merchants. However, it continued the monopoly of the company overtrade in tea and trade with
China. Moreover, it authorized the Local Governments in India to impose taxes on persons. They
could also punish the people for not paying taxes.
Sources:
https://egyankosh.ac.in/bitstream/123456789/20342/1/Unit-23.pdf
Indian Polity by M. Laxmikanth, 6th Edition, Chapter -1

25. Answer: B
Explanation:
● Option (b) is correct: Indian Councils Act of 1892 increased the functions of legislative councils
and gave them the power of discussing the budget and addressing questions to the executive.
It increased the number of additional (non-official) members in the Central and provincial
legislative councils, but maintained the official majority in them. It provided for the nomination of
some non-official members of the Central Legislative Council by the viceroy on the
recommendation of the provincial legislative councils and the Bengal Chamber of Commerce, and
that of the provincial legislative councils by the Governors on the recommendation of the district
boards, municipalities, universities, trade associations, zamindars and chambers. ‘The act made a
limited and indirect provision for the use of election in filling up some of the non-official seats
both in the Central and provincial legislative councils. The word “election” was, however, not used
in the Act. The process was described as nomination made on the recommendation of certain
bodies.
Source:
Indian Polity by M. Laxmikanth, 6th Edition, Chapter 1-Historical Background

26. Answer: C
Explanation:
● Option (c) is correct: Food and Agriculture portfolio were held by Dr. Rajendra Prasad in the
Interim Government formed in the year 1946. After the Indian Independence in 1947, Dr.
Rajendra Prasad became the first President of India. Baldev Singh and Jawaharlal Nehru held the
portfolios of Defence and External Affairs and Commonwealth respectively.
Source: UPSC Civil Services Examination (Prelims) 2006

27. Answer: D
Explanation:
● Option (d) is correct: The 97th Constitutional Amendment Act of 2011 gave a Constitutional
status and protection to co-operative societies. In this context, it made the following three
changes in the Constitution:
o It made the right to form co-operative societies a fundamental right (Article 19).
o It included a new Directive Principle of State Policy on promotion of co-operative societies
(Article 43-B).
o It added a new Part IX-B in the Constitution which is entitled as ‘The Co-operative Societies’
(Articles 243-ZH to 243-ZT).
The new Part IX-B contains various provisions to ensure that the co-operative societies in the country
function in a democratic, professional, autonomous and economically sound manner. It empowers
the Parliament in respect of multi-state cooperative societies and the state legislatures in respect of
other co-operative societies to make the appropriate law.
Source: Indian Polity by M Laxmikanth 6th edition, Chapter 3

For More Study Material, Visit: studyiq.com Page no.


11
SIP POLITY TEST 01

28. Answer: C
Explanation:
The features or principles of parliamentary government in India are:
● Statement 1 is correct: Nominal and Real Executives – The President is the nominal executive
(de jure executive or titular executive) while the Prime Minister is the real executive (de facto
executive). Thus, the President is head of the State, while the Prime Minister is head of the
government. Article 74 provides for a council of ministers headed by the Prime Minister to aid and
advise the President in the exercise of his functions. The advice so tendered is binding on the
President.
● Statement 2 is incorrect: The British system is based on the doctrine of the sovereignty of
Parliament, while the Parliament is not supreme in India and enjoys limited and restricted
powers due to a written Constitution, federal system, judicial review and fundamental rights.
● Statement 3 is correct: Collective Responsibility is the bedrock of the principle of parliamentary
government. The ministers are collectively responsible to the Lok Sabha (Article 75(3)). They act
as a team, and swim and sink together. The principle of collective responsibility implies that the
Lok Sabha can remove the ministry (i.e., council of ministers headed by the prime minister) from
office by passing a vote of no confidence.
● Statement 4 is correct: Double Membership – The ministers are members of both the legislature
and the executive. This means that a person cannot be a minister without being a member of the
Parliament. The Constitution stipulates that a minister who is not a member of the Parliament for
a period of six consecutive months ceases to be a minister.
Source: Indian Polity by M Laxmikanth 6th edition, Chapter 12

29. Answer: C
Explanation:
Article 21 declares that no person shall be deprived of his life or personal liberty except according to
procedure established by law. This right is available to both citizens and non-citizens.
● Statement 1 is incorrect: In the famous Gopalan case (1950), the Supreme Court has taken a
narrow interpretation of Article 21. It held that the protection under Article 21 is available only
against arbitrary executive action and not from arbitrary legislative action. This means that the
State can deprive the right to life and personal liberty of a person based on a law. This is because
of the expression ‘procedure established by law’ in Article 21, which is different from the
expression ‘due process of law’ contained in the American Constitution. Hence, the validity of a
law that has prescribed a procedure cannot be questioned on the ground that the law is
unreasonable, unfair or unjust. Secondly, the Supreme Court held that the ‘personal liberty’
means only liberty relating to the person or body of the individual.
● Statement 2 is incorrect: In the Maneka Gandhi case (1978), the Supreme Court overruled its
judgement in the Gopalan case by taking a wider interpretation of Article 21. It ruled that the
right to life and personal liberty of a person can only be deprived by a law provided the
procedure prescribed by that law is reasonable, fair and just. In other words, it has introduced
the American expression ‘due process of law’. In effect, the protection under Article 21 should
be available not only against arbitrary executive action but also against arbitrary legislative action.
Further, the court held that the ‘right to life’ as embodied in Article 21 is not merely confined to
animal existence or survival but it includes within its ambit the right to live with human dignity
and all those aspects of life which go to make a man’s life meaningful, complete and worth living.
It also ruled that the expression ‘Personal Liberty’ in Article 21 is of the widest amplitude and it
covers a variety of rights that go to constitute the personal liberties of a man.
Source: Indian Polity by M Laxmikanth 6th edition, Chapter 7

For More Study Material, Visit: studyiq.com Page no.


12
SIP POLITY TEST 01

30. Answer: B
Explanation:
● Statement 1 is correct: The Charter Act of 1853 introduced, for the first time, local
representation in the Indian (Central) Legislative Council. Of the six new legislative members of
the Governor General’s council, four members were appointed by the local (provincial)
governments of Madras, Bombay, Bengal and Agra.
● Statement 2 is correct but statement 3 is incorrect: It introduced an open competition system of
selection and recruitment of civil servants. The covenanted civil service was, thus, thrown open to
the Indians also. Accordingly, the Macaulay Committee (the Committee on the Indian Civil
Service) was appointed in 1854. Lord Macaulay was appointed the President of the Committee.
However, the exams were to be held only in England. It was Montagu Chelmsford reforms of
1919 which recommended holding of civil service examinations simultaneously in India and
England.
Source:
Indian Polity by M. Laxmikanth, 6th Edition, Chapter 1- Historical Background

31. Answer: C
Explanation:
Indian Councils Act of 1909 is also known as Morley-Minto Reforms (Lord Morley was the then
Secretary of State for India and Lord Minto was the then Viceroy of India)
● Statement 1 is incorrect: The Government of India Act of 1919 also known as Montagu-
Chelmsford Reforms relaxed the central control over the provinces by demarcating and
separating the central and provincial subjects. The central and provincial legislatures were
authorized to make laws on their respective list of subjects. However, the structure of government
continued to be centralized and unitary.
● Statement 2 is correct: Indian Councils Act of 1909 provided (for the first time) for the
association of Indians with the executive councils of the Viceroy and Governors. Satyendra
Prasad Sinha became the first Indian to join the Viceroy’s executive council. He was appointed as
the Law Member.
● Statement 3 is incorrect: The Government of India Act of 1919 introduced, for the first time,
bicameralism and direct elections in the country. Thus, the Indian legislative council was replaced
by a bicameral legislature consisting of an Upper House (Council of State) and a Lower House
(Legislative Assembly). The majority of members of both the Houses were chosen by direct
election. Government of India Act of 1935 introduced bicameralism in six out of eleven
provinces. Thus, the legislatures of Bengal, Bombay, Madras, Bihar, Assam and the United
Provinces were made bicameral consisting of a legislative council (upper house) and a legislative
assembly (lower house). However, many restrictions were placed on them
● Statement 4 is incorrect: The Government of India Act of 1919 provided for the establishment
of a public service commission. Hence, a Central Public Service Commission was set up in 1926
for recruiting civil servants. This was done on the recommendation of the Lee Commission on
Superior Civil Services in India (1923- 24).
● Statement 5 is correct: Morley-Minto Reforms considerably increased the size of the legislative
councils, both Central and provincial. The number of members in the Central legislative council
was raised from 16 to 60. The number of members in the provincial legislative councils was not
uniform. It retained official majority in the Central legislative council, but allowed the provincial
legislative councils to have non-official majority.
Source:
Indian Polity by M. Laxmikanth, 6th Edition, Chapter 1-Historical Background

32. Answer: D
Explanation:

For More Study Material, Visit: studyiq.com Page no.


13
SIP POLITY TEST 01

● Statement 1 is incorrect: It was in 1934 that the idea of a Constituent Assembly for India was
put forward for the first time by M.N. Roy, a pioneer of communist movement in India. In 1935,
the Indian National Congress (INC), for the first time, officially demanded a Constituent Assembly
to frame the Constitution of India. In 1938, Jawaharlal Nehru, on behalf the INC declared that ‘the
Constitution of free India must be framed, without outside interference, by a Constituent
Assembly elected on the basis of adult franchise’. The demand was finally accepted in principle by
the British Government in what is known as the ‘August Offer’ of 1940. In 1942, Sir Stafford Cripps,
a Member of the Cabinet, came to India with a draft proposal of the British Government on the
framing of an independent Constitution to be adopted after the World War II. The Cripps Proposals
were rejected by the Muslim League, which wanted India to be divided into two autonomous
states with two separate Constituent Assemblies. Finally, a Cabinet Mission1 was sent to India.
While it rejected the idea of two Constituent Assemblies, it put forth a scheme for the Constituent
Assembly which more or less satisfied the Muslim League
● Statement 2 is incorrect: Sir B.N. Rau was appointed as the constitutional advisor (Legal advisor)
to the Constituent Assembly. The Constituent Assembly held its first meeting on December 9,
1946. The Muslim League boycotted the meeting and insisted on a separate state of Pakistan. The
meeting was, thus, attended by only 211 members. Dr. Sachchidananda Sinha, the oldest member,
was elected as the temporary President of the Assembly, following the French practice. Later, Dr.
Rajendra Prasad was elected as the President of the Assembly. Similarly, both H.C. Mukherjee and
V.T. Krishnamachari were elected as the Vice-Presidents of the Assembly.
Source:
Indian Polity by M. Laxmikanth, 6th Edition, Chapter 2- Making of the Constitution

33. Answer: D
Explanation:
● Option (d) is correct: Constitutions seek to limit and regulate the exercise of political power.
Constitutional government is by definition limited government. It means that the government
is run in accordance with rules and principles that are binding on all political actors. As a result,
by separating or dividing power, the constitutional government helps to constrain its unfettered
exercise. A constitutional government establishes a framework for the government to be both
accountable and representative.
Source: UPSC Civil Services Examination (Prelims) 2021

34. Answer: C
Explanation:
● Option (c) is correct: In the Minerva Mills case (1980), the Supreme Court held that the Indian
Constitution is founded on the ‘bedrock of the balance between the Fundamental Rights and
the Directive Principles’. They together constitute the core of commitment to social revolution.
They are like two wheels of a chariot, one no less than the other. To give absolute primacy to one
over the other is to disturb the harmony of the Constitution. This harmony and balance between
the two is an essential feature of the basic structure of the Constitution. The goals set out by the
Directive Principles have to be achieved without the abrogation of the means provided by the
Fundamental Rights’.
Source: Indian Polity by M Laxmikanth 6th edition, Chapter 8

35. Answer: B
Explanation:
● Statement 1 is incorrect: Article 348(1) of the Constitution of India provides that all proceedings
in the Supreme Court and in every High Court shall be in English language until Parliament by
law otherwise provides.

For More Study Material, Visit: studyiq.com Page no.


14
SIP POLITY TEST 01

● Statement 2 is incorrect: Article 348 (2) provides that the Governor of the state may, with the
previous consent of the President, authorize the use of the Hindi language or any other language
used for any official purpose of the state, in the proceedings of the High Court having its
principal seat in that State provided that decrees, judgments or orders passed by such High Courts
shall be in English. The Official Language Act, 1963 reiterates this and provides under Section 7
that the use of Hindi or official language of a State in addition to the English language may be
authorized, with the consent of the President of India, by the Governor of the State for the
purpose of judgments, decrees etc. made by the High Court for that State. No law has been made
in this regard by the Parliament so far. Therefore, English continues to be the language for all the
proceedings of the Supreme Court.
● Statement 3 is correct: The language of all courts subordinate to High Courts generally remains
the same as the language on the commencement of the Civil Procedure Code 1908, till the state
government determines. There are two provisions regarding the use of language in subordinate
courts. Under Section 137 of the Code of Civil Procedure, the language of the district courts shall
be similar to the language of the act. The state government has the power to declare any regional
language as an alternative for the proceedings of the court.
Sources:
https://doj.gov.in/use-of-hindi-and-regional-languages/
https://www.advocatekhoj.com/library/bareacts/codeofcivilprocedure/137.php

36. Answer: B
Explanation:
Recently, a batch of petitions seeking criminalisation of marital rape, has been filled in the Delhi High
Court. In response to it the Union government has replied that it is considering a ‘constructive
approach’ towards criminalizing it and had sought suggestions from various stakeholders. The petition
seeks to amend the criminal law, which includes Section 375 (rape) of the Indian Penal Code (IPC).
● Statement 1 is incorrect: The grounds for “marital immunity” for rape prosecution have emerged
from the patriarchal discourse in society. According to which, a husband cannot be guilty of a rape
committed upon his lawful wife because she has given up herself in this kind to her husband by
their mutual matrimonial consent and contract, which she cannot retract. Section 375 of the
Indian Penal Code exempts forceful sexual intercourse by a man with his own wife from the
offence of rape, provided the wife is above 15 years of age, also known as the ‘marital rape
exception’.
● Statement 2 is correct: The Justice J. S. Verma Committee was constituted in the aftermath of
the horrific Nirbhaya gang rape in 2012. While some of its recommendations helped shape the
Criminal Law (Amendment) Act, 2013, its most radical suggestions, including on marital rape,
were never implemented.
Source:
http://www.legalservicesindia.com/article/2500/Does-Section-375-of-IPC-Include-Marital-Rape.html

37. Answer: B
Explanation:
● Statement 1 is incorrect: Lord Stanley, the President of the Board of Control introduced a bill for
the 'Better Government' of India which became an Act of Parliament in August 1858. It abolished
the East India Company, and the Government of India passed from the hands of the English East
India Company to the crown. The armed forces of the company were transferred to the crown.
The Board of control and court of Directors were abolished. Their place was taken by the Secretary
of State of India and his India Council. They were to govern India in the name of her majesty. The
Government of India in its dealings with England was guided by the directions laid down by the
Secretary of State in Council. All matters concerning legislation, land revenue, public works,
railways, jobs, new expenditure and policies were rigidly scrutinised and controlled by the

For More Study Material, Visit: studyiq.com Page no.


15
SIP POLITY TEST 01

Secretary of State. The Rules and Regulations made in India by the secretary of state were to be
laid on the table of the House of Commons. The Governor General became known from now as
the Viceroy or Crown's representative. In matters of policy and its execution the viceroy was
increasingly reduced to a subordinate position in relation to the British Government. The
Government of India was finally directly controlled from London. However, it did not alter in any
substantial way the system of Government that prevailed in India.
● Statement 2 is correct: The Government of India Act of 1858 promised equal and impartial
protection under law to all Indians, besides equal opportunities in government services
irrespective of race or creed. It was also promised that old Indian rights, customs and practices
would be given due regard while framing and administering the law.
● Statement 3 is incorrect: The Charter Act of 1813 asserted the British Crown’s sovereignty over
the Company’s territories in India. The Government of India Act of 1858 provided that India,
henceforth, was to be governed by, and in the name of, Her Majesty. It changed the designation
of the Governor-General of India to that of Viceroy of India. He (Viceroy) was the direct
representative of the British Crown in India. Lord Canning, thus, became the first Viceroy of India
Sources:
Indian Polity by M. Laxmikanth, 6th Edition, Chapter 1- Historical Background
https://egyankosh.ac.in/bitstream/123456789/20342/1/Unit-23.pdf

38. Answer: A
Explanation:
Government of India Act of 1935 marked a second milestone towards a completely responsible
government in India. It was a lengthy and detailed document having 321 Sections and 10 Schedules.
● Statement 1 is incorrect: Government of India Act of 1935 introduced bicameralism in six out of
eleven provinces. Thus, the legislatures of Bengal, Bombay, Madras, Bihar, Assam and the
United Provinces were made bicameral consisting of a legislative council (upper house) and a
legislative assembly (lower house). However, many restrictions were placed on them.
● Statement 2 is incorrect: The Act provided for the adoption of dyarchy at the Centre.
Consequently, the federal subjects were divided into reserved subjects and transferred subjects.
However, this provision of the Act did not come into operation at all.
There were to be three subject lists—the Federal Legislative List, the Provincial Legislative List and
the Concurrent Legislative List. Residuary legislative powers were subject to the discretion of the
Governor-General. Even if a bill was passed by the federal legislature, the governor-general could veto
it, while even Acts assented to by the governor-general could be disallowed by the King-in-Council
● Statement 3 is correct: It abolished dyarchy in the provinces and introduced ‘provincial
autonomy’ in its place. The provinces were allowed to act as autonomous units of administration
in their defined spheres. Moreover, the Act introduced responsible Governments in provinces,
that is, the Governor was required to act with the advice of ministers responsible to the provincial
legislature. This came into effect in 1937 and was discontinued in 1939. Provincial governments
could borrow money on their own security. Provinces were freed from “the superintendence,
direction” of the secretary of state and governor-general. Provinces henceforth derived their legal
authority directly from the British Crown. Provinces were given independent financial powers and
resources.
Question Hack: There are currently just six states in India with a bicameral setup. These are more or
less the same states in which bicameralism was introduced in 1935. So, all 11 states are not
plausible.
Sources:
Indian Polity by M. Laxmikanth, 6th Edition, Chapter 1 Historical Background;
Brief history of Modern India Spectrum, Chapter 26 Constitutional, Administrative and Judicial
Developments, pdf no 543,544

For More Study Material, Visit: studyiq.com Page no.


16
SIP POLITY TEST 01

39. Answer: A
Explanation:
● Statement 1 is correct: In the three Presidency towns of Calcutta, Madras and Bombay there were
two existing judicial systems for administering Justice that is, the Supreme Court and the Sadar
Diwani and Sadar Nizamat Adalat. This sort of judicial administration was inconvenient for the
inhabitants of the Presidencies. Ultimately, this problem was resolved by the British Parliament
by enacting the Indian High Courts Act, 1861. The Indian High Courts Act, 1861, abolished the
Supreme Court and Sadar Adalat's in the Presidencies and the Act also empowered the Crown to
issue Letter's Patent under the great Seal of the United Kingdom, to erect and establish High Court
of Judicature at Calcutta, Madras and Bombay.
● Statement 2 is incorrect: Government of India Act of 1935 provided for the establishment of a
Federal Court, which was set up in 1937. It provided for the establishment of a Reserve Bank of
India to control the currency and credit of the country. 9. It provided for the establishment of not
only a Federal Public Service Commission, but also a Provincial Public Service Commission and
Joint Public Service Commission for two or more provinces.
Sources:
Indian Polity by M. Laxmikanth, 6th Edition, Chapter 1 Historical Background;
https://www.legalserviceindia.com/legal/article-7006-the-emergence-and-evolution-of-high-courts-
in-india-before-the-constitution-of-india-came-into-effect.html

40. Answer: B
Explanation:
● Statement 1 is correct: To achieve the export target of $2 trillion by 2030, India is going the whole
hog on free trade agreements (FTAs). India is negotiating FTAs with countries such as the European
Union, Canada, the U.K., and Israel. There is no mechanism for any role of the Indian Parliament
in the ratification of free trade agreements in India. India negotiates most FTAs behind closed
doors with very little information about the objectives and processes followed and negligible
scrutiny. The Commerce Ministry is the nodal body dealing with FTAs and on its website provides
the bare minimum information about FTA negotiations. However, this is not the case in other
countries with whom India is negotiating such an FTA. In the U.K., for example, there are several
robust mechanisms that foster a certain degree of transparency in the FTA negotiations.
Furthermore, there are institutional apparatuses that enable the scrutiny of the actions of the
executive, during and after the signing of the FTA.
● Statement 2 is incorrect: Free Trade Agreement is a pact between two or more countries, where
the objective is to remove trade barriers of imports and exports and ensure hassle-free trade
relations as far as possible. Under a free trade agreement, countries offer preferential trade
terms, tariff concessions to each other. Free trade agreements could be bilateral or multilateral.
For instance, India has signed an agreement called South Asian Free Trade Area with South Asian
countries including Nepal, Sri Lanka, Bangladesh, Bhutan etc. In addition to free trade agreements,
India has also signed six preferential trade agreements (PTAs). The main difference between the
both is that an FTA is comprehensive covering various areas, while a PTA is limited to only trade
in goods and seeks only a tariff elimination in terms of a margin of preference.
● Statement 3 is correct: According to the leaked draft IP chapter put out by an international trade
portal bilaterals.org, the UK is seeking continuous extension of patent period and rights through
small tweaks in the drug, known as evergreening. To put simple, Evergreening is the practice of
companies filing for patent extensions by making minor changes to their drugs just before the
patent expires at 20 years. A clause allowing patent evergreening in the India-UK free trade
agreement will harm India’s generics industry and the UK’s healthcare service that is dependent
on Indian drugs. The draft has also proposed to prohibit pre-grant oppositions, which means India
will not be able to stop challenges to weak/ invalid patents until after they have been granted.
Sources:

For More Study Material, Visit: studyiq.com Page no.


17
SIP POLITY TEST 01

https://www.thehindu.com/opinion/op-ed/quest-for-transparency-in-fta-
negotiations/article66144419.ece
https://www.business-standard.com/article/economy-policy/india-uk-trade-deal-leaked-draft-ip-
chapter-suggests-stringent-patent-rule-122110201512_1.html
https://www.business-standard.com/podcast/economy-policy/what-is-a-free-trade-agreement-
122110500122_1.html

41. Answer: A
Explanation:
● Statement 1 is correct: The World Bank’s classification of the world’s economies is based on
estimates of gross national income (GNI) per capita. The GNI is gross national income converted
to international dollars using purchasing power parity rates. The most current World Bank Income
classifications by GNI per capita (updated July 1 of every year) are as follows:
o Low income: $1,025 or less
o Lower middle income: $1,026 to $4,035
o Upper middle income: $4,036 to $12,475
o High income: $12,476 or more
Low- and middle-income economies are usually referred to as developing economies, and the
Upper Middle Income and the High Income are referred to as Developed Countries. In its latest
classification (2020-21), the World Bank has classified India as a lower-middle-income country.
● Statement 2 is incorrect: Recently, China’s status as a ‘developing country’ at the World Trade
Organization (WTO) has become a contentious issue with a number of countries raising concerns
over the upper middle-income nation deriving benefits reserved for developing countries under
WTO norms. The WTO has not defined ‘developed’ and ‘developing’ countries and therefore
member countries are free to announce whether they are ‘developed’ or ‘developing’. However,
given the rise in China’s per capita income to become an upper middle-income country
according to the World Bank and the country’s alleged use of unfair trade practices such as
preferential treatment for state enterprises, data restrictions and inadequate enforcement of
intellectual property rights, a number of nations have called on China to either refrain from
seeking benefits available to developing countries or forego its classification as a developing
country altogether.
● Statement 3 is incorrect: The IMF refers to the classification of countries as Advanced and
Emerging and Developing Economies. The main criteria used by the IMF in country classifications
are:
o Gross domestic product (GDP) per capita.
o Export diversification.
o degree of integration into the global financial system: This includes both a country's volume
of international trade and its adoption of and participation in international financial
institutions.
The IMF uses either sums or weighted averages of data for individual countries.
Sources:
https://www.a4id.org/policy/understanding-the-developeddeveloping-country-taxonomy/
https://indianexpress.com/article/explained/at-wto-china-a-developing-country-why-many-nations-
are-raising-concerns-7716778/#:~:text=(AP%2FFile)-
,China's%20status%20as%20a%20'developing%20country'%20at%20the%20World%20Trade,develo
ping%20countries%20under%20WTO%20norms

42. Answer: B
Explanation:
On August 20, 1917, the British Government declared, for the first time, that its objective was the
gradual introduction of responsible Government in India. The Government of India Act of 1919 was

For More Study Material, Visit: studyiq.com Page no.


18
SIP POLITY TEST 01

thus enacted, which came into force in 1921. This Act is also known as Montagu-Chelmsford Reforms
(Montagu was the Secretary of State for India and Lord Chelmsford was the Viceroy of India).
● Statement 1 is correct: It extended the principle of communal representation by providing
separate electorates for Sikhs, Indian Christians, Anglo-Indians and Europeans. It granted
franchise to a limited number of people on the basis of property, tax or education.
● Statement 2 is incorrect: Government of India Act of 1935 provided for the establishment of an
All-India Federation consisting of provinces and princely states as units. The Act divided the
powers between the Centre and provincial units in terms of three lists–Federal List (for Centre,
with 59 items), Provincial List (for provinces, with 54 items) and the Concurrent List (for both,
with 36 items). Residuary powers were given to the Viceroy. However, the federation never came
into being as the princely states did not join it.
● Statement 3 is correct: Montagu-Chelmsford Reforms separated, for the first time, provincial
budgets from the Central budget and authorized the provincial legislatures to enact their
budgets. It provided for the appointment of a statutory commission to inquire into and report on
its working after ten years of its coming into force.
● Statement 4 is incorrect: The Government of India Act of 1919 further divided the provincial
subjects into two parts-transferred and reserved. The transferred subjects were to be
administered by the Governor with the aid of Ministers responsible to the legislative council. The
reserved subjects, on the other hand, were to be administered by the Governor and his
executive council (not ministers) without being responsible to the legislative council. This dual
scheme of governance was known as ‘dyarchy’-which means double rule. However, this
experiment was largely unsuccessful.
Source:
Indian Polity by M. Laxmikanth, 6th Edition, Chapter 1-Historical Background

43. Answer: C
Explanation:
● Statement 1 is correct: The Government of India has released the entire amount of GST
compensation payable to States up to 31st May, 2022 by releasing an amount of Rs.86,912
crores.
● Statement 2 is incorrect: Goods and Services Tax was introduced in the country w.e.f. 1st July,
2017 and States were assured for compensation for loss of any revenue arising on account of
implementation of GST as per the provisions of the GST (Compensation to States) Act, 2017 for a
period of five years. For providing compensation to States, Cess is being levied on certain goods
[not all goods and services] and the amount of Cess collected is being credited to Compensation
Fund. The compensation amount to be paid from the compensation fund is arrived at by levying
cess on top of the highest tax slab on luxury, demerit and sin goods. Compensation to States is
being paid out of the Compensation Fund w.e.f. 1st July, 2017
● Statement 3 is correct: The GST Council in the September meeting last year decided to extend the
compensation cess period till March 2026, but the collection will be used “purely to repay the
back to back loans taken between 2020-21 and 2021-22" and not to further compensate states,
as per the Finance Ministry.
Source:
https://www.livemint.com/news/india/extend-gst-compensation-to-states-by-5-years-or-raise-
share-chhattisgarh-to-fm-11656408406923.html
https://pib.gov.in/PressReleasePage.aspx?PRID=1829777
https://www.thehindu.com/business/gst-compensation-cess-levy-extended-till-march-
2026/article65563659.ece

44. Answer: B
Explanation:

For More Study Material, Visit: studyiq.com Page no.


19
SIP POLITY TEST 01

● Statement 1 is correct and Statement 2 is incorrect: The Indian Constitution adopted universal
adult franchise as a basis of elections to the Lok Sabha and the state legislative assemblies. Every
citizen who is not less than 18 years of age has a right to vote without any discrimination of
caste, race, religion, sex, literacy, wealth and so on. The voting age was reduced to 18 years from
21 years in 1989 by the 61st Constitutional Amendment Act of 1988. The introduction of
universal adult franchise by the Constitution-makers was a bold experiment and highly remarkable
in view of the vast size of the country, its huge population, high poverty, social inequality and
overwhelming illiteracy.14 Universal adult franchise makes democracy broad-based, enhances the
self-respect and prestige of the common people, upholds the principle of equality, enables
minorities to protect their interests and opens up new hopes and vistas for weaker sections.
● Statement 3 is correct: Though the Indian Constitution is federal and envisages a dual polity
(Centre and states), it provides for only a single citizenship, that is, the Indian citizenship. In
countries like the USA, on the other hand, each person is not only a citizen of the USA, but also a
citizen of the particular state to which he belongs. Thus, he owes allegiance to both and enjoys
dual sets of rights–one conferred by the National government and another by the state
government. In India, all citizens irrespective of the state in which they are born or reside enjoy
the same political and civil rights of citizenship all over the country and no discrimination is made
between them.
Source: Indian Polity by M Laxmikanth 6th edition, Chapter 3

45. Answer: A
Explanation:
● Statement 1 and 2 are correct: Recently, the WTO’s dispute settlement panel had ruled that
India, by subsidising sugar producers, was breaking rules framed under the General Agreement
on Tariffs and Trade (GATT) which govern international trade. In 2019, Australia, Brazil, and
Guatemala complained against India at the WTO arguing that subsidies offered by the Indian
government to sugar producers were against the rules governing international trade. They argued
that these subsidies, which include both domestic subsidies as well as export subsidies, exceed
the limits imposed by WTO trade rules. According to WTO rules, subsidies cannot exceed 10% of
the total value of sugar production. These countries believe that subsidies offered by India have
led to increased production of sugar and caused the price of sugar to drop significantly in the
global market.
● Statement 3 is incorrect: The Appellate Body of WTO, set up in 1995, is the standing committee
that presides over appeals against judgments passed in trade-related disputes brought by WTO
members. The reports Appellate Body of WTO, once adopted by the WTO’s disputes settlement
body (DSB), are final and binding on the parties. Like the General Council, the DSB is composed
of representatives of all WTO Members.
Sources:
https://www.thehindu.com/business/Economy/explained-why-is-india-challenging-wto-verdict-on-
sugar/article38275758.ece
https://indianexpress.com/article/explained/explained-the-wtos-dispute-settlements-mechanism-is-
all-but-dead-this-is-why-india-should-worry-6158502/
https://www.wto.org/english/tratop_e/dispu_e/disp_settlement_cbt_e/c3s1p1_e.htm

46. Answer: A
Explanation:
● Statement 1 is correct: The Russian invasion of Ukraine was the largest conventional military
attack seen since World War II with Russia conducting a multi-pronged offensive across the
country. The Russian military pummeled wide areas in Ukraine with airstrikes and has conducted
massive rocket and artillery bombardment resulting in massive casualties.

For More Study Material, Visit: studyiq.com Page no.


20
SIP POLITY TEST 01

● Statement 2 is incorrect: The NATO-Russia Council was set up two decades ago but full meetings
paused when Russia annexed Ukraine’s Crimean Peninsula in 2014. Recently, it has met in 2022
since, the last time in July 2019.
● Statement 3 is incorrect: The first Minsk Agreement was signed in September 2014 and the
second agreement in March 2015. They were aimed at enabling a permanent peaceful solution
to the geopolitical tension between Russia and Ukraine. Note- Ukraine is not a NATO member
country. Example: the first Minsk Agreement was signed in Belarus in September 2014.
Signatories to the agreement were Russia, Ukraine, Organisation for Security and Cooperation in
Europe (OSCE) and the pro-Russia leaders from Donetsk and Luhansk.
Source:
https://www.thehindu.com/news/international/nato-russia-in-high-level-talks-as-ukraine-tensions-
simmer/article38251248.ece

47. Answer: C
Explanation:
● Statement 1 is incorrect: The Indian Independence Act was passed in 1947. The act created two
new independent dominions of India and Pakistan. Pakistan was split into Pakistan and East
Pakistan which is now Bangladesh. The Bengal and Punjab provinces were partitioned between
the two new countries. The Act repealed the use of ‘Emperor of India' as a title for the British
Crown and ended all existing treaties with the princely states. Lord Mountbatten continued as
Governor-General and Jawaharlal Nehru was appointed India's first Prime Minister, Muhammad
Ali Jinnah became Pakistan's Governor-General and Liaquat Ali Khan its Prime Minster. It provided
for the governance of each of the dominions and the provinces by the Government of India Act of
1935, till the new Constitutions were framed. The dominions were however authorized to make
modifications in the Act.
● Statement 2 is correct: The Nehru Report of 1928 confined itself to British India, as it envisaged
the future link-up of British India with the princely states on a federal basis. It recommended
dominion status on lines of self-governing dominions as the form of government desired by
Indians. It rejected separate electorates which had been the basis of constitutional reforms so far;
instead, a demand for joint electorates with reservation of seats for Muslims at the Centre and in
provinces where they were minority in proportion to the Muslim population there with right to
contest additional seats. It provided for nineteen fundamental rights including equal rights for
women, right to form unions, and universal adult suffrage.
● Statement 3 is correct: Indian Independence Act, 1947 abolished the office of the Secretary of
State for India and transferred his functions to the Secretary of State for Commonwealth Affairs.
It abolished the office of Viceroy and provided, for each dominion, a governor-general, who was
to be appointed by the British King on the advice of the dominion cabinet
Sources:
Indian Polity by M. Laxmikanth, 6th Edition, Chapter 1-Historical Background;
Spectrum Brief history of Modern India by Rajiv Ahir, Chapter 18- Simon Commission and the Nehru
Report
https://www.parliament.uk/about/living-
heritage/evolutionofparliament/legislativescrutiny/parliament-and-
empire/collections1/collections2/1947-indian-independence-act/ ;
https://www.legislation.gov.uk/ukpga/1947/30/pdfs/ukpga_19470030_en.pdf

48. Answer: C
Explanation:
● Statement 1 is incorrect: Assam and Meghalaya share an 885-km-long border. As of now, there
are 12 points of dispute along their borders. The Assam-Meghalaya border dispute are the areas
of Upper Tarabari, Gazang reserve forest, Hahim, Langpih, Borduar, Boklapara, Nongwah,

For More Study Material, Visit: studyiq.com Page no.


21
SIP POLITY TEST 01

Matamur, Khanapara-Pilangkata, Deshdemoreah Block I and Block II, Khanduli and Retacherra.
Meghalaya was carved out of Assam under the Assam Reorganisation Act, 1971, a law that it
challenged, leading to disputes.
● Statement 2 is incorrect: A major point of contention between Assam and Meghalaya is the
district of Langpih in West Garo Hills bordering the Kamrup district of Assam. Langpih was part of
the Kamrup district during the British colonial period but post-Independence, it became part of
the Garo Hills and Meghalaya. Assam considers it to be part of the Mikir Hills in Assam.
Meghalaya has questioned Blocks I and II of the Mikir Hills -now Karbi Anglong region - being part
of Assam. Meghalaya says these were parts of erstwhile United Khasi and Jaintia Hills districts.
Both Assam and Meghalaya have constituted border dispute settlement committees. It has been
decided to set up two regional committees to resolve the border disputes in a phased manner and
various aspects will be considered while resolving the border dispute.
Question Hack: Assam was ruled by the Ahom kingdom for centuries even before the arrival of the
British and was incorporated within British India in 1833. So, it is very unlikely that it would have
been carved out of another state like Meghalaya.
Source:
https://indianexpress.com/article/explained/assam-meghalaya-border-dispute-recent-killing-
impacts-8285785/

49. Answer: C
Explanation:
The Drafting Committee was the most important committee among all the committees of the
Constituent Assembly. It was set up on August 29, 1947, just 14 days after the independence. The
committee was entrusted with the task of preparing a draft of the new Constitution. It consisted of
seven members, namely:
1. Dr. B.R. Ambedkar (Chairman)
2. N. Gopalaswamy Ayyangar
3. Alladi Krishnaswamy Ayyar
4. Dr. K.M. Munshi
5. Syed Mohammad Saadullah
6. N. Madhava Rau (He replaced B.L. Mitter who resigned due to ill-health)
7. T.T. Krishnamachari (He replaced D.P. Khaitan who died in 1948)
The committee prepared the first draft of the Constitution of India, which was published in February,
1948. The Drafting Committee prepared a second draft, which was published in October, 1948. The
Drafting Committee took less than six months to prepare its draft. In all it sat for 141 days.
• Option 1 is correct: Dr. B.R. Ambedkar was the chairman of the Drafting committee.
• Option 2 is incorrect: Pattabhi Sitaramayya was the chairman of the House Committee which was
a minor committee.
• Options 3 and 4 are correct: N. Gopalaswamy Ayyangar and Alladi Krishnaswamy Ayyar were the
members of the Drafting committee.
Source: Indian Polity by M. Laxmikanth, 6th Edition: Chapter 2- Making of the Constitution

50. Answer: D
Explanation:
The motion on Draft Constitution was declared as passed on November 26, 1949, and received the
signatures of the members and the president. Out of a total 299 members of the Assembly, only 284
were actually present on that day and signed the Constitution. This is also the date mentioned in the
Preamble as the date on which the people of India in the Constituent Assembly adopted, enacted and
gave to themselves this Constitution.
● Option (a) is correct: The Constitution was adopted on November 26, 1949.

For More Study Material, Visit: studyiq.com Page no.


22
SIP POLITY TEST 01

● Option (b) is correct: January 26 was specifically chosen as the ‘date of commencement’ of the
Constitution because of its historical importance. It was on this day in 1930 that Purna Swaraj
day was celebrated, following the resolution of the Lahore Session (December 1929) of the INC.
● Option (c) is correct: With the commencement of the Constitution, the Indian Independence Act
of 1947 and the Government of India Act of 1935, with all enactments amending or
supplementing the latter Act, were repealed. The Abolition of Privy Council Jurisdiction Act
(1949) was however continued.
● Option (d) is incorrect: The adopted constitution contained Preamble, 395 Articles and 8
Schedules. 9th schedule was added to the Constitution in 1951 and 10th schedule containing anti-
defection laws was added in 1985.
Source: Indian Polity by M. Laxmikanth, 6th Edition: Chapter 2: Making of the Constitution

For More Study Material, Visit: studyiq.com Page no.


23

You might also like